PT41,S1,Q12 (LR) Forum

Prepare for the LSAT or discuss it with others in this forum.
Post Reply
cardsfan04

New
Posts: 30
Joined: Sun Feb 13, 2011 5:51 pm

PT41,S1,Q12 (LR)

Post by cardsfan04 » Sat Jul 23, 2011 11:22 am

I eliminated A and B on the first scan and C on the second scan. I ended up getting this right, but without a ton of confidence because D and E seem very similar to me. I think I know why D is wrong, but want to make sure.

In the stimulus, the author argues for a reason why violent offenders became non-violent. In D, it isn't looking at violent offenders, so the high-nutrient diet might not be the cause of those offenders being non-violent. They might already be non-violent and just happen to eat high-nutrient diets.

Am I at least in the ballpark for this?

When I did it, I created a contrapositive:

VB=violent behavior
PN= poor nutrition

PN-->VB
-VB-->-PN

That seems consistent with D though, so I was thrown for a loop. Is it wrong to assume an if/then relationship when it isn't explicitly stated? I think this thinking is what made this difficult for me.

Sloth Hero

Bronze
Posts: 233
Joined: Sun Apr 17, 2011 2:32 pm

Re: PT41,S1,Q12 (LR)

Post by Sloth Hero » Sat Jul 23, 2011 5:39 pm

I think were on the same page for E, but I'll flush it out, and the conditional and contrapositive should be for the conclusion only.

E) Is obviously correct because it acts as the "control". Since the stimi argues for a link between poor nutrition and violent behavior, it is crucial that there be two groups, one with a changed diet, and one with the original diet, ceteris paribus. Since one group changed and the other didn't, it seems likely that a link between nutrition and violence exists.

If it weren't the case that there was a control group, then maybe over the 4 month period their violence decreased due to the panoptical forces of the prison, or whatever (Bentham :D)

D) Is wrong because it doesn't set up as strong of an experiment as 'E', and it doesn't rule out the possibility that non-violent people might have a disposition to eat nutritious food-- but not that there is an actual link between nutrition and behavior.

cardsfan04

New
Posts: 30
Joined: Sun Feb 13, 2011 5:51 pm

Re: PT41,S1,Q12 (LR)

Post by cardsfan04 » Mon Jul 25, 2011 10:12 am

OK. That makes sense. Thanks for the response.

Post Reply

Return to “LSAT Prep and Discussion Forum”